This is a weakening question, we know because the question stem says: Which one of the following, if true, most undermines the author’s prediction?
This is a nice and short stimulus; some unexpected heavy rainfall has filled reservoirs and streams, and therefore the author predicts that it won’t be necessary to ration water this summer. The assumption underlying the argument is a conditional; if the reservoirs and streams are full, then water rationing won’t be necessary. The correct answer is going to undermine this hidden premise that the reservoirs and streams being full is enough for water rationing not to be needed. Like many weakening questions, there is a lot of ways to come at this argument, and therefore POE is the best way to approach the answer choices. Let’s take a look at them:
Answer Choice (A)This is a poor answer because it relies on inductive reasoning (what happened in the past will happen in the future), and the author specifies that the reservoirs and streams being filled by heavy rainfall is a recent and unexpected phenomenon. What happened in previous years is not relevant to the support for the prediction.
Answer Choice (B) This is a very weak answer. For one, it admits that only a small part of the city’s water supply comes from this underground supply; you would expect that it would have little effect on whether water rationing will be required. All this answer does is introduce another supply in addition to the full reservoirs we’ve been told about, which would strengthen the prediction that water rationing won’t be necessary.
Correct Answer Choice (C)This answer does exactly what we identified in the stimulus; it undermines the sufficiency of full reservoirs for a lack of water rationing. If the actual transportation of the water to individuals is the issue, then a surplus of water isn’t enough to prevent water shortages among customers, and therefore it is entirely possible water rationing will be necessary even if the water supply is full.
Answer Choice (D) This answer doesn’t help us because we don’t know much about the relation of temperature to water usage. If this instead said “long-range weather forecasts predicts an extremely hot summer, which may increase water usage” then it might be helpful. But as it is stated, D fails to weaken the prediction.
Answer Choice (E) This answer is similar to A in that it relies on the inductive assumption that what happens in most years will happen in future ones. Even if this answer guaranteed that there would be less rain in the summer, for all we know the full reservoirs could supply the city for another year without any rain.
This is a weakening question: Which one of the following, if true, most seriously weakens the argument?
The first thing we learn from the stimulus is that there is this species of spider, the painted spider, and they have stickier webs than the other spiders that share their habitat. Stickier webs are better at trapping specifically flying insects, spiders hunt for food by trapping insects, and therefore the argument concludes the spider is a more successful predator than its competitors.
The overall structure of this argument is fairly simple; there is this way in which a particular kind of spider differs from others, and since this would be helpful for preying on flying insects, we conclude that in virtue of this one factor the painted spider is a better predator than its competitors. The problem with this argument is we know very little; we know very little about other possible differences between the painted spider and other spiders, about the overlap between other spiders and the painted spiders competitors, and about what portion of the prey in this habitat are specifically flying insects. The right answer is going to introduce some information that exploits one of these gaps in the argument. Let’s eliminate the answers that fail to do so:
Answer Choice (A) This addresses the issue of what portion of the prey in the habitat are specifically the flying insects which would be susceptible to the painted spider’s stickier webs. The problem is that it gives us almost no real information; “not all” could mean one single non-flying insect or almost all of the insects. Without more information, this answer is entirely compatible with both supporting this argument (a low number of non-flying insects) or weakening it (a large number of non-flying insects), and therefore does not by itself seriously weaken the argument.
Answer Choice (B) All this answer does is tell us that certain kinds of insects would be unlikely to be trapped by non-sticky webs, and therefore gives us a reason to think the painted spider might have a real advantage over other spiders. If this were a strengthening question that might be meaningful, but for us this does the opposite of what we want.
Answer Choice (C) This is just an irrelevant fact. Whether the spider paralyzes or kills its victim doesn’t bear any clear relation to the impact of its sticky webs on its success as a predator compared to its competitors.
Correct Answer Choice (D) This provides a downside to the painted spider’s stickier webs. Even if they are more efficient at trapping insects that fly into them, if they are more visible maybe much less insects fly into them, so that the painted spider is actually a much worse predator than other spiders. At the very least, this weakens the main support for the argument’s conclusion, that the painted spider’s stickier webs give it a plain and simple advantage.
Answer Choice (E) E introduces web size to equation, and does so in a really unhelpful way. If this said that the webs were significantly smaller than other spiders and therefore caught less insects, that might help us weaken the conclusion that painted spiders are better predators, but as stated this answer does nothing for us.
This is a strengthening question, which we know because the question stem asks: Which one of the following, if true, most strongly supports the conclusion drawn by the entomologists?
The stimulus tells us that a few years ago some entomologists (insect scientists) introduced a fungus to oak forests to get rid of gypsy moths which eat oak leaves. The fungus is poisonous specifically to the caterpillars of these moths, and in the years since the population of both the caterpillars and moths has declined where the fungus was introduced. The entomologists conclude with a hypothesis that it was the fungus which caused the moth decline.
Like many strengthening questions this one involves a causal hypothesis; the passage gives us a correlation between the introduction of fungus and the decline of the moths and caterpillars, and we are supposed to find the answer which provides more justification for the entomologists’ hypothesis that this correlation really is a causal relation. With that in mind, let’s go about eliminating some of these answers:
Correct Answer Choice (A) This answer gives us a subset of gypsy moths with which to compare the effects on the other susceptible moths. The only difference between the rest of the gypsy moths and this strain which we are told about is that the subset is immune to the poisonous fungus. This makes them a really good control population to test our hypothesis. Since the only known factor by which the two groups differs is whether they are affected by the fungus, and the resistant group is growing in size while the moths as a whole decline, this is good evidence that it is the fungus which has caused the overall decline of the moths and caterpillars.
Answer Choice (B) This question introduces other insects to the equation, and you should immediately recognize that no mention is made of them in the entomologists’ hypothesis. Other insect groups are irrelevant to the phenomenon of gypsy moth decline.
Answer Choice (C) This answer introduces another possible cause of the gypsy moth decline, predation. Considering we are trying to support the fungus hypothesis, this answer is if anything weakens what we want to strengthen, and an increase of moths and caterpillars following a drop in predation does nothing to explain the overall significant decline which we are interested in.
Answer Choice (D) This answer introduces an entirely new phenomenon, a decline in oak tree populations caused by air pollution and acid rain. This might be a good answer if this were a weakening question, since the moths feed on oak trees and therefore the decline in oak trees would be an alternative explanation for the decline of moths. But since this is a strengthening question this answer does the opposite of what we are looking for, and is consequently incorrect.
Answer Choice (E) Just as A introduced a control population which was immune to the fungus, E introduces a control population where the fungus has not been introduced. The problem is that where A suggested the fungus as a cause because relative to the general moth population the fungus-immune moths seemed to be doing well, here the moths seem to be declining regardless of whether the fungus is present. This shows that the correlation between population and fungus decline is very weak, and therefore weakens the fungus hypothesis which we want to strengthen.
This is a strengthening question, as the stem asks us: Which one of the following, if true, most strongly supports Ping’s conclusion?
The passage takes the form of a dialogue between Winston and Ping over a transit authority’s attempt to operate without a budget deficit. We know from the question stem that we are interested in Ping’s conclusion. The first sentence appears a bit more complicated than it really is; it throws a lot of information and negations at us but what should really jump out at you is the “unless”. If we convert this unless into a conditional, we get something along the lines of “if the PTA does not cut late night services, then it cannot fulfill its mandate.” Whenever you see a sentence in an argument begin with “since” and then transition after a comma into a second clause, you should pick up the hint that the author is about to give a conclusion after introducing a premise. “Since [premise] is true, [conclusion].” Winston does exactly this, and gives us the conclusion of his argument that the cuts would reduce the deficit and should be made. As a cherry on top, he ends with another premise telling us that fare increases take two years, eliminating another possible remedy at least in the immediate future.
Ping responds to Winston by pointing out that cutting late-night service would not only affect fares purchased during that period, but would also decrease fares during the day, insofar as there are “those riders” who do round trips with one ride in the day and one during the late-night period. It makes sense when you think about it, if you work a job that finishes late, you might still commute to work during the day. If you can’t take the transit home after work at night, then you are more likely to find another way to commute during the day as well. It is a bit difficult to identify the conclusion in this argument, because it actually includes two. It is tempting to consider the last sentence the conclusion that the question stem told us about, because it begins with the indicator “thus”. However, this is actually a sub-conclusion which supports Ping’s overall position which he states in his opening sentence; the proposed service cuts might cost more than they would save. The correct answer will support Ping’s position that cutting late night service could lose the PTA money. Let’s look at the answers:
Correct Answer Choice (A) What this answer does is give us a bit more insight into those riders that Ping mentions. Ping’s argument as stated assumes those riders represent a significant enough proportion of ridership that the loss of their fares might be greater than the savings of cutting late-night services. For all we know, these riders could be a very small group, or not even exist. What A does is fill in this gap in the argument, by explicitly telling us that those riders make up almost a quarter of all PTA round trips. While 23% isn’t as strong support as a higher percentage, it’s important not to let this throw you off; the proof that those riders exist at all adds support to Ping’s conclusion.
Answer Choice (B) This answer tells us the riders’ opinion on cutting late-night services. Unfortunately, we don’t care what the riders think. Ping’s conclusion is simply a prediction that if the cuts happen, an undesirable result (loss of money) might occur. This is true regardless of the opinions of riders.
Answer Choice (C) C like B introduces completely irrelevant information to Ping’s conclusion. Winston mentions that it is a long process to get a fare increase approved, and all this does is strengthen that a fare increase is not a viable solution for the PTA to avoid a budget deficit. But fare increases have no bearing on Ping’s argument because it is a prediction about late night service cuts.
Answer Choice (D) Whichever LSAC writer did this question sure loves percentages, and we get another one thrown at us. Once again this answer is irrelevant to Ping’s argument. The relative severity of the deficit does not affect Ping’s claim about the potential for service cuts to lose the PTA money.
Answer Choice (E) This answer doesn’t give us enough information, and if anything hurts Ping’s argument by introducing another cost associated with late-night services which might outweigh the fares earned from those riders. While it tells us that the cash bonuses are “significant”, we don’t know how many bus drivers there are or even whether the late-night service is composed of mostly buses as opposed to trains, streetcars, or what.
Here we have a weakening question, as the stem asks: Which one of the following, if true, most weakens the patent reviewers’ argument?
Our stimulus begins with some context about a patent application for a genetically engineered lab mouse that got denied. In support of their decision, the patent reviewers argued that the mouse was a new variety of animal and they aren’t allowed to approve patents for new animal varieties. This argument is a case of rule application, a conditional rule (new animal variety → no patent) is used to justify a particular conclusion (no patent for the lab mouse). It is important to note that we have not been given an explanation of what makes an animal a new variety. We know the mouse has been genetically engineered, but not whether this means it really is a new variety. That the mouse is a new variety is an assumption upon which the reviewers’ argument depends. Since there are many ways to weaken an argument, we can’t assume that this assumption will play a role in the correct answer, but we should take note of it regardless. Let’s look at the answer choices.
Correct Answer Choice (A) This answer choice completely undermines the reviewers’ argument. If the new variety rule only applies to farm animals, than it can’t be used to support denying the laboratory mouse patent.
Answer Choice (B) Who cares? This answer choice is completely irrelevant to the patent reviewers’ decision and reasoning.
Answer Choice (C) All this tells us is that the patent reviewers are consistent in their application of the rule. It does nothing to undermine their reasoning for why the rule supports denying the lab mouse patent.
Answer Choice (D) The rule that the reviewers’ cited as their reason specifically applies to animal varieties. It is entirely consistent with their reasoning that they would approve patents which don’t violate the rule. D is incorrect.
Answer Choice (E) Similar to C this just gives us some extra context that the patent reviewers have consistently applied the rule to past animal varieties. What we want is a reason why the rule shouldn’t apply to the lab mouse, which A gives us.
This is a most strongly supported question, as the question stem asks: If both Monroe’s conclusion and the evidence on which he bases it are correct, they would provide the strongest support for which one of the following?
From the question stem alone we already know someone named Monroe is going to be giving us a conclusion supported by some evidence. The first sentence gives us a phenomenon; Monroe ate at the Tip-Top Restaurant and enjoyed the meals, but afterwards became ill each time. He must have really enjoyed it to go back after being sick two times in a row! Since we have started with a phenomenon, Monroe getting sick every time he eats at Tip-Top, we should expect a causal hypothesis as to why this correlation is occurring. The next sentence gives us some more detail about this correlation. Monroe ate three different meals at Tip-Top, but each one had hot peppers. So this Monroe getting sick every time he eats at Tip-Top correlation is also more specifically a Monroe getting sick every time he eats hot peppers correlation. Where the first sentence probably made you think, “Maybe you should try a new restaurant, Monroe!”, this new information has properly altered your hypothesis to, “Maybe ask them to skip the hot peppers next time, Monroe!”. The final sentence tells us that Monroe agrees and gives us his conclusion, his hypothesis for why he kept getting sick; it was the hot peppers causing him to feel sick after all three meals. He probably should have been able to figure that out after two meals, but good for Monroe!
Having now read the stimulus, and remembering that we are supposed to assume Monroe’s hot pepper hypothesis is correct, our job is to select the answer that is most strongly supported based off of it. Let’s take a look at the answer choices:
Answer Choice (A) What this answer choice does is confuse sufficiency with necessity. Just because hot peppers are enough for Monroe to feel sick, doesn’t mean they are required for him to be sick. Maybe he’s also allergic to pine nuts and the all-you-can-eat specials includes a really nice Pesto pasta! If you chose this answer, you might want to review conditional logic.
Correct Answer Choice (B) This is the correct answer because it does what A tries to do without its error of confusing enough with required. Where A said Monroe can eat anything without hot peppers and not get sick, B correctly infers that if Monroe ate a different dish with hot peppers he would have still gotten sick, because hot peppers are enough for him to be ill.
Answer Choice (C) Same issue as A, this question incorrectly assumes that hot peppers are the only possible cause of Monroe feeling ill.
Answer Choice (D) We need to assume that we just don’t know about Monroe’s eating history for this answer to follow from our stimulus.
Answer Choice (E) This should be clearly wrong. We know nothing about Monroe’s restaurant history outside of Tip-Top. Maybe Monroe is really bad at noticing when an ingredient makes him feel sick and he’s actually eaten hot peppers 20 times at other restaurants and gotten sick, and only now after three meals at Tip-Top made the connection.
This is a most strongly supported question, since the question stem says: Which one of the following conclusions is most strongly supported by the results of the experiments?
As you should have noticed in the question stem, this stimulus involves experiments. We should therefore be on the lookout for a phenomenon. The first sentence begins with the conditional indicator only, and from it we learn that some strains of tobacco are resistant to tobacco mosaic virus; important to note that resistant here doesn’t mean they resist infection, but rather they resist developing symptoms and becoming diseased. This resistance is our phenomenon. The first of the experiments gives us a correlation. Tobacco plants infected with the mosaic virus who resisted its effects had increased levels of salicylic acid, while non-resistant plants had no increase in their levels. Resistance to the virus correlates with more salicylic acid. The second experiment greatly strengthens the correlation by artificially raising the salicylic levels of half of 50 non-resistant plant, with every high salicylic level plant resisting the disease and every normal plan succumbing to it.
If we were to summarize this stimulus, it is essentially: phenomenon, correlation, even stronger correlation. What we never receive in this stimulus is a conclusion, or more specifically in this case, a hypothesis. If we pre-phrase, we would expect a conclusion to this information to infer causation from correlation, and conclude that the increased salicylic acid at least partly causes the mosaic resistance. We should be on the lookout for an answer that introduces this hypothesis, while eliminating any answers that have little to no support in the stimulus. Let’s take a look at our options:
Answer Choice (A) None of the experiments involved salicylic acid removing symptoms, but only preventing them. Our correlation is between salicylic acid and resistance, not healing.
Correct Answer Choice (B) This answer does exactly what we should have predicted in our pre-phrasing; it introduces the expected hypothesis that salicylic acid at least partly explains why some plants are able to resist the mosaic virus.
Answer Choice (C) This information is not only not supported, but likely false based on the stimulus. The salicylic acid is described as naturally occurring in the plants, and the difference between the resistant and non-resistant plants is whether there was an increase in salicylic acid.
Answer Choice (D) Remember we are told about an increase in the salicylic levels following infection; if the plant is uninfected the stimulus suggests that its salicylic acid levels will be normal, and maybe even indistinguishable from those of non-resistant plants.
Answer Choice (E) This answer might be appealing since it does emphasize the correlation between an increase in salicylic acid and resistance to the virus. The problem is that the experiments involved artificially injecting plants to raise their salicylic levels, and we have no information about whether it is possible increase the plant’s production of the acid.
This is a weakening question, since we are asked: Which one of the following, if true, most seriously weakens the argument?
This stimulus is relatively straightforward. We are first told that a portion of economically useful raw materials are both nonrenewable and in limited supply on Earth. So we can’t make more of these resources and we don’t have such a big supply that we would never run out. Following this premise we get a sentence beginning with the conclusion indicator therefore and the conditional indicator unless. If we translate this conditional conclusion, we should end up with something along the lines of: no materials can be obtained outside earth → unable to accomplish what is now accomplished with those materials (referential phrase: the useful, non-renewable, and limited supply materials).
When we are faced with a weakening question involving a standard argument, we should always consider if the argument relies on a large assumption. In this case, we are concluding that if we can’t find more of these materials, we won’t be able to do what we use them for right now. The major assumption here is that these materials are required for what we do with them. For example, just because we currently use gas to produce electricity doesn’t mean that gas is required to produce electricity. An easy way to weaken this argument is to explicitly state that these materials aren’t required, and could be replaced with alternatives. We should be on the lookout for an answer choice that does this. Let’s see what we get:
Answer Choice (A) The argument we are trying to weaken is about the portion of economically useful materials which aren’t renewable. Even if there are renewable resources, unless they are viable alternatives to our non-renewables, we don’t care about them.
Answer Choice (B) This answer choice just suggests that the conditional conclusion of our argument will be triggered, since it is hard to get resources outside of earth. This does nothing to weaken or even strengthen the argument.
Correct Answer Choice (C) This contradicts the assumption we identified in the stimulus. If these limited nonrenewable resources can be replaced with alternatives, then even if we can’t find more of them we can still keep doing what we are doing with them. In short, this says the resources aren’t required, which is exactly what the argument assumes they are.
Answer Choice (D) Who cares? The argument we are trying to weaken has a conditional conclusion; it makes a prediction about what will happen in a particular case. The worthiness of what the resources are being used for is irrelevant to whether it will be impossible to continue should they run out.
Answer Choice (E) This answer fills in one of the assumptions the argument makes; that we will ever run out of these resources just because there is a limited supply of them. For that reason, if anything, all this answer does is strengthen what we want to weaken.
This is a most strongly supported question, as it asks: The statements above provide the most support for which one of the following?
The stimulus begins by informing us that there is a correlation between the distance animals travel and the size of their groups, and their diets. This just means what an animal eats is consistently related to how far it travels and what kind of group it lives with. So maybe herd animals that travel long distances usually live off mostly grass or something; we don’t get any details, just that the relationship exists. Next we learn that diet itself correlates with the animals’ faces and teeth. And that’s it! With a short MSS stimulus like this, we should be thinking about what kind of inferences we can make with so little information. In this case, if there is a connection between traveling/group behavior and diet, and between diet and face/teeth, then maybe you can somewhat reliably predict how an animal travels/groups just based off what kind of face/teeth it has. Let’s take a look at the answer choices:
Answer Choice (A) We’ve been told that diet and travel correlate, but not how. This answer requires a lot of assumptions about the actual details of the correlation.
Answer Choice (B) We’ve been told that diet correlates with face and teeth shape and size, but not the details of this correlation and certainly nothing about how overall size correlates with diet.
Answer Choice (C) This is a very specific detail. To infer it just off the two sentences we were given would require a ton of assumptions.
Answer Choice (D) What should really signal that this answer is wrong is “all that is needed”. That is a very strong claim to draw from our two sentences.
Correct Answer Choice (E) Compare the “all that is needed” of D to the mere “can” of this answer choice. Exactly as we predicted in our pre-phrase, this answer makes the relatively small inference that if diet correlates with teeth and face, and diet also correlates with how an animal travels, then there might be a correlation between teeth and face, and how an animal travels.
This is a strengthening question, as the question stem asks: Which one of the following, if true, most strengthens the argument?
The stimulus starts by telling us that the rate at which children aged 4 and under use safety seats has doubled within the last 8 years. Good to hear! The next sentence begins with the claim that this increased usage of safety seats has prevented child fatalities. This is the conclusion of our argument. The sentence continues by conceding that it is true that child fatalities have risen by 10% during the same period that safety seat usage has increased, but attributes this to a 20% increase in the number of serious accident.
This author’s conclusion is that the safety seats are working—the increase in their usage has prevented fatalities. However, the fact that child fatalities have actually increased during the exact same period is a big problem for this position. The author’s defends his safety seats are working conclusion by pointing to the increase in accidents—it’s not that safety seats aren’t preventing fatalities, its just that there are a lot more opportunities for people to die. The issue with this defense is we have no idea much of the increase in serious automobile accidents involve children. Let’s look at the answer choices and see which answer strengthens the author’s position:
Answer Choice (A) Irrelevant. We are interested in how the number of children aged 4 and under killed could have increased in the same period that safety seat usage increased and prevented fatalities.
Correct Answer Choice (B) This strengthens the argument by making the 20% increase in serious accidents a much better defense of the safety seats are working conclusion. If serious accidents went up 20% while the proportion involving children remained constant, but child fatalities only increased by 10%, it seems that the children are dying at a lower rate than would be expected, which in turn suggests that the safety seats really are having an effect.
Answer Choice (C) If they are in cars the same amount of time then nothing has changed in terms of their chance of being involved in accidents.
Answer Choice (D) We are concerned with children aged 4 and under as a whole, the varying usage of seatbelts between subsets of children aged 4 and under has no impact on our argument
Answer Choice (E) If the adult fatalities increased at the same rate as the child fatalities while the safety seat usage increased, it suggests the doubling of safety seats had no impact on child fatalities.